Cell Adaptations, Cell Injury and Cell Death

Lakukan tugas rumah & ujian kamu dengan baik sekarang menggunakan Quizwiz!

*The answer is A.* Metaplastic change TO a squamous epithelium may occur in the bronchi in cigarette smokers.

Metaplastic change TO a squamous epithelium may occur in the following circumstances: A Bronchi in cigarette smokers. B Appendix in appendicitis. C Skin in wound healing D Upper oesophagus in oesophagitis.

*The answer is C.* The type of cell death is apoptosis, which requires caspase activation.

The above is a picture of epithelium in a patient with a viral infection. The type of cell death seen in the cells (by arrows) is characterized by which of the following? A. Cell swelling B. Marked inflammation C. Caspase activation D. Bax deactivation E. BCL-2 activation

*The answer is C.* Nuclear fragmentation is an irreversible damage.

Under the light microscope, which of the following signifies a cell has progressed to irreversible damage? A. Plasma membrane blebs B. Cytoplasmic vacuoles C. Nuclear fragmentation D. Cytoplasmic eosinophilia E. Fatty change

Caseous Necrosis

What morphologic type of necrosis is seen in this kidney and what might cause it? (Tricky! Look carefully at its consistency...)

Proliferative Macrophage (at top), activated fibroblast and a small vessel. This is granulation tissue.

What stage of wound healing is seen in the photo? What are the arrow heads pointing to?

*The answer is D.* Coagulative necrosis initially maintains cell outlines.

Which is the type of necrosis that initially maintains cell outlines? A. liquefactive B. caseous C. gangrenous D. coagulative

*The answer is D.* Swelling of the ER is reversible.

Which of the following would be the most likely cellular change to be found if it were possible to examine microscopic sections of the heart in a patient whose myocardium was entirely "saved" by immediate thrombolytic (clot dissolving) therapy? (A) Karyorrhexis (B) Pyknosis (C) Karyolysis D) Swelling of the endoplasmic reticulum (hydropic change)

*The answer is B.* Hutchinson-Gilford progeria is a rare genetic disease characterized by early cataracts, hair loss, atrophy of the skin, osteoporosis, and atherosclerosis. This phenotype gives the impression of premature aging in children. Progeria is one of many diseases caused by mutations in the human lamin A gene (LMNA). Lamins are intermediate filament proteins that form a fibrous meshwork beneath the nuclear envelope. Defective lamin A is thought to make the nucleus unstable, leading to cell injury and death. Mutations in the other genes are not linked to Hutchinson-Gilford progeria syndrome. Diagnosis: Progeria

A 10-year-old girl presents with advanced features of progeria (patient shown in the image). This child has inherited mutations in the gene that encodes which of the following types of intracellular proteins? (A) Helicase (B) Lamin (C) Oxidase (D) Polymerase (E) Topoisomerase

*The answer is C.* The liver is one of the few organs in the human body that can partially regenerate. This is a form of compensatory hyperplasia. The stimuli to hepatocyte mitotic activity cease when the liver has attained its normal size. Hepatocytes can reenter the cell cycle and proliferate to regenerate the liver; they do not just hypertrophy (increase in size). Apoptosis is single cell death and frequently occurs with viral hepatitis. Dysplasia is disordered epithelial cell growth that can be premalignant. Hydropic change, or cell swelling, does not produce regeneration. Steatosis (fatty change) can lead to hepatomegaly, but not as a regenerative process. It is the result of toxic/metabolic hepatocyte injury.

A 16-year-old boy sustained blunt trauma to his abdomen when he struck a bridge abutment at high speed while driving a motor vehicle. Peritoneal lavage shows a hemoperitoneum, and at laparotomy, a small portion of the left lobe of the injured liver is removed. Two months later, a CT scan of the abdomen shows that the liver has nearly regained its size before the injury. Which of the following processes best explains this CT scan finding? A Apoptosis B Dysplasia C Hyperplasia D Hydropic change E Steatosis

*The answer is E.* The liver is one of the few organs in the human body that can partially regenerate. This is a form of compensatory hyperplasia. The stimuli to hepatocyte mitotic activity cease when the liver has attained its normal size. Apoptosis is single cell death and frequently occurs with viral hepatitis. Dysplasia is disordered epithelial cell growth that can be premalignant. Fatty change can lead to hepatomegaly; this is not a regenerative process, but is the result of toxic hepatocyte injury. Hydropic change, or cell swelling, does not produce regeneration. Hepatocytes can reenter the cell cycle and proliferate to regenerate the liver; they do not just increase in size.

A 16-year-old boy sustained blunt trauma to the abdomen when the vehicle he was driving struck a bridge abutment at high speed. Peritoneal lavage shows a hemoperitoneum, and at laparotomy, a small portion of the left lobe of the liver is removed because of the injury. Several weeks later, a CT scan of the abdomen shows that the liver has nearly regained its size before the injury. Which of the following processes best explains this CT scan finding? (A) Apoptosis (B) Dysplasia (C) Fatty change (D) Hydropic change (E) Hyperplasia (F) Hypertrophy (G) Metaplasia

*The answer is E.* There is an increase in the breast lobules under hormonal influence, primarily progesterone with pregnancy, to provide for lactation.

A 19-year-old woman gives birth to her first child. She begins breast feeding the infant. She continues breast feeding for almost a year with no difficulties and no complications. Which of the following cellular processes that began in the breast during pregnancy allowed her to nurse the infant for this period of time? A Stromal hypertrophy B Epithelial dysplasia C Steatocyte atrophy D Ductal epithelial metaplasia E Lobular hyperplasia

*The answer is D.* Breast lobules have an increased number of cells under hormonal influence (mainly progesterone) to provide for normal lactation. Ductal metaplasia in the breast is a pathologic process. Epithelial dysplasia denotes disordered growth and maturation of epithelial cells that may progress to cancer. Accumulation of fat within cells is a common manifestation of sublethal cell injury or, uncommonly, of inborn errors in fat metabolism. The breast stroma plays no role in lactation and may increase with pathologic processes.

A 20-year-old woman breastfeeds her infant. On examination, her breasts are slightly increased in size. Milk can be expressed from both nipples. Which of the following processes that occurred in her breasts during pregnancy enables her to breastfeed the infant? A Ductal metaplasia B Epithelial dysplasia C Intracellular lipid deposition D Lobular hyperplasia E Stromal hypertrophy

*The answer is A.* The ongoing pressure load of the systemic hypertension led to myocardial fiber hypertrophy and a heart that increased in size. Renal failure often leads to hypertension, and vice versa.

A 20-year-old woman had Goodpasture syndrome which progressed to chronic renal failure. She is 165 cm tall and weighs 55 kg. She now has blood pressure measurements in the range of 150/90 to 180/110 mm Hg, but does not regularly take medications. Laboratory studies show her blood urea nitrogen is over 100 mg/dL and she requires chronic dialysis. A chest x-ray shows an enlarged heart. The size of her heart is most likely to be the result of which of the following processes involving the myocardial fibers? A Hypertrophy B Fatty infiltration C Hyperplasia D Fatty degeneration E Edema

*The answer is E.* Polymorphonuclear leukocytes (segmented neutrophils) rapidly accumulate at sites of injury. They are loaded with acid hydrolases and are capable of digesting dead cells. A localized collection of these inflammatory cells may create an abscess with central liquefaction (pus). Liquefactive necrosis is also commonly seen in the brain. Caseous necrosis (choice A) is seen in necrotizing granulomas. Fat necrosis (choice C) is typically encountered in patients with acute pancreatitis. Fibrinoid necrosis (choice D) is seen in patients with necrotizing vasculitis.

A 22-year-old construction worker sticks himself with a sharp, rusty nail. Within 24 hours, the wound has enlarged to become a 1-cm sore that drains thick, purulent material. This skin wound illustrates which of the following morphologic types of necrosis? (A) Caseous necrosis (B) Coagulative necrosis (C) Fat necrosis (D) Fibrinoid necrosis (E) Liquefactive necrosis

*The answer is D.* The increase in uterine size is primarily the result of an increase in the size of myometrial smooth muscle cells. The endometrium also increases in size, mainly via hyperplasia, but it remains as a thin lining to the muscular wall and does not contribute as much to the change in size. There is little stroma in myometrium and a greater proportion in endometrium, so stroma contributes a smaller percentage to the gain in size than muscle. The vessels are a minor but essential component in this increase in size, but not the largest component.

A 22-year-old woman becomes pregnant. A fetal ultrasound examination at 13 weeks' gestation shows her uterus measures 7 × 4 × 3 cm. At delivery of a term infant, her uterus measures 34 × 18 × 12 cm. Which of the following cellular processes has contributed most to the increase in her uterine size? A Endometrial glandular hyperplasia B Myometrial fibroblast proliferation C Endometrial stromal hypertrophy D Myometrial smooth muscle hypertrophy E Vascular endothelial hyperplasia

*The answer is A.* There is an apoptotic cell (arrow) that is shrunken and has been converted into a dense eosinophilic mass. There is a surrounding inflammatory reaction with cytotoxic lymphocytes. This pattern is typical of apoptosis. Caspase activation is a universal feature of apoptosis, regardless of the initiating cause. Apoptosis induced in recipient cells from donor lymphocytes occurs with graft-versus-host disease. Lipases are activated in enzymatic fat necrosis. Reduced ATP synthesis and increased glycolysis occur when a cell is subjected to anoxia, but these changes are reversible. Lipid peroxidation occurs when the cell is injured by free radicals.

A 22-year-old woman with leukemia undergoes bone marrow transplantation and receives a partially mismatched donor marrow. One month later, she has a scaling skin rash. A skin biopsy is obtained, and on microscopic examination, it has the cellular change shown in the figure. This change most likely results from which of the following biochemical reactions? A Activation of caspases B Elaboration of lipases C Increase in glycolysis D Peroxidation of lipids E Reduction of ATP synthesis

*The answer is A.* (A) CORRECT. Gangrenous necrosis is described. Occlusive peripheral atherosclerotic vascular disease is typical for diabetes mellitus. Many arteries are involved, reducing collateral flow. This is diabetic gangrene.

A 60-year-old woman has noted a dark red-black appearance to her great toe and second and third toes of her left foot for the past month. On physical examination, the toes are cold and have no sensation to touch. The dorsalis pedis and posterior tibial pulses are not palpable on the left. A transmetatarsal amputation is performed. These findings are most typical for a patient with which of the following conditions? A Diabetes mellitus B Gout C Blunt force trauma D AIDS E Rheumatoid arthritis

*The answer is D.* The metabolism of CCl4 is a model system for toxicologic studies. CCl4 is first metabolized via the mixed function oxygenase system (P450) of the liver to a chloride ion and a highly reactive trichloromethyl free radical. Like the hydroxyl radical, this radical is a potent initiator of lipid peroxidation, which damages the plasma membrane and leads to cell death. The other choices are not involved in the formation of the trichloromethyl free radical in liver cells. Diagnosis: Hepatic failure, hepatotoxicity

A 24-year-old woman accidentally ingests carbon tetrachloride (CCl4) in the laboratory and develops acute liver failure. Which of the following cellular proteins was directly involved in the development of hepatotoxicity in this patient? (A) Acetaldehyde dehydrogenase (B) Alcohol dehydrogenase (C) Glucose-6-phosphate dehydrogenase (D) Mixed function oxygenase (E) Superoxide dismutase

*The answer is E.* Carbon tetrachloride and acetaminophen are well-studied hepatotoxins. Each is metabolized by cytochrome P450 of the mixed-function oxidase system, located in the smooth endoplasmic reticulum. These hepatotoxins are metabolized differently, and it is possible to relate the subsequent evolution of lethal cell injury to the specific features of this metabolism. Acetaminophen, an important constituent of many analgesics, is innocuous in recommended doses, but when consumed to excess it is highly toxic to the liver. The metabolism of acetaminophen to yield highly reactive quinones is accelerated by alcohol consumption, an effect mediated by an ethanol-induced increase in cytochrome P450. Diagnosis: Hepatotoxicity, necrosis

A 24-year-old woman with chronic depression ingests a bottle of acetaminophen tablets. Two days later, she is jaundiced (elevated serum bilirubin) and displays symptoms of encephalopathy, including impairment in spatial perception. In the liver, toxic metabolites of acetaminophen are generated by which of the following organelles? (A) Golgi apparatus (B) Mitochondria (C) Nucleus (D) Peroxisomes (E) Smooth endoplasmic reticulum

*The answer is A.* The description is that of a typical infarct with vascular occlusion following embolization from vegetations of infective endocarditis.

A 26-year-old man has had a high fever for the past 2 days. On exam he has a heart murmur. Echocardiography shows destruction of the aortic valve by large, irregular vegetations. Staphylocoaccus aureus is cultured from his blood. He develops left upper quadrant pain. Abdominal CT shows a wedge-shaped 1.5 x 3 cm splenic lesion with base on the capsule. The splenic lesion is most likely to result from which of the following cellular abnormalities? A Coagulative necrosis B Abscess formation C Metaplasia D Caseous necrosis E Liquefactive necrosis

*The answer is A.* The most common form of atrophy follows reduced functional demand. For example, after immobilization of a limb in a cast as treatment for a bone fracture, muscle cells atrophy, and muscular strength is reduced. The expression of differentiation genes is repressed. On restoration of normal conditions, atrophic cells are fully capable of resuming their differentiated functions; size increases to normal, and specialized functions, such as protein synthesis or contractile force, return to their original levels. Ischemic necrosis (choice D) is typically a complication of vascular insufficiency. Irreversible injury to skeletal muscle (choice E) would be an unlikely complication of bone fracture. Diagnosis: Atrophy, bone fracture

A 28-year-old woman is pinned by falling debris during a hurricane. An X-ray film of the leg reveals a compound fracture of the right tibia. The leg is immobilized in a cast for 6 weeks. When the cast is removed, the patient notices that her right leg is weak and visibly smaller in circumference than the left leg. Which of the following terms best describes this change in the patient's leg muscle? (A) Atrophy (B) Hyperplasia (C) Metaplasia (D) Ischemic necrosis (E) Irreversible cell injury

*The answer is B.* Reduced workload causes cell to shrink through loss of cell substance, a process called atrophy. The cells are still present, just smaller. Aplasia refers to lack of embryonic development; hypoplasia describes poor or subnormal development of tissues. Dystrophy of muscles refers to inherited disorders of skeletal muscles that lead to muscle fiber destruction, weakness, and wasting. Hyaline change (hyalinosis) refers to a nonspecific, pink, glassy eosinophilic appearance of cells.

A 29-year-old man sustains a left femoral fracture in a motorcycle accident. His leg is placed in a plaster cast. After his left leg has been immobilized for 6 weeks, the diameter of the left calf has decreased in size. This change in size is most likely to result from which of the following alterations in his calf muscles? A Aplasia B Atrophy C Dystrophy D Hyalinosis E Hypoplasia

*The answer is C.* After removal of part of the liver, remaining liver can undergo hyperplasia to compensate. Hepatocytes are stable cells that are able to re-enter the cell cycle.

A 3-year-old child has developed hepatic failure. The left lobe of an adult donor liver is used as an orthotopic transplant. A year later, the size of each liver in donor and recipient is greater than at the time of transplantation. Which of the following cellular alterations is most likely to explain this phenomenon? a) Metaplasia b) Hypertrophy c) Hyperplasia d) Anaplasia e) Neoplasia

*The answer is C.* Reduced workload causes shrinkage of cell size because of loss of cell substance, a process called atrophy. Aplasia refers to lack of embryonic development; hypoplasia describes poor or subnormal development. Dystrophy of muscles refers to inherited disorders of skeletal muscles that lead to muscle weakness and wasting.

A 30-year-old man sustains a left femoral fracture in a skiing accident, and his leg is placed in a plaster cast. After the leg has been immobilized for several weeks, the diameter of the left calf has decreased. This change is most likely to result from which of the following alterations in the calf muscles? (A) Aplasia (B) Hypoplasia (C) Atrophy (D) Dystrophy

*The answer is B.* Fat necrosis is seen with trauma to the breast, and her lawyer will make good use of that documentation. The pattern of multiple injuries of differing ages at different sites suggests abuse.

A 30-year-old woman is claiming in a civil lawsuit that her husband has abused her for the past year. A workup by her physician reveals a 2 cm left breast mass. There is no lymphadenopathy. No skin lesions are seen, other than a bruise to her upper arm. A needle biopsy of the breast mass is performed. On microscopic examination, the biopsy shows fat necrosis. This biopsy result is most consistent with which of the following etiologies? A Physiologic atrophy B Breast trauma C Lactation D Radiation injury E Hypoxic injury

*The answer is C.* Psoriasis is a disease of the dermis and epidermis that is characterized by persistent epidermal hyperplasia. It is a chronic, frequently familial disorder that features large, erythematous, scaly plaques, commonly on the dorsal extensor cutaneous surfaces. There is evidence to suggest that deregulation of epidermal proliferation and an abnormality in the microcirculation of the dermis are responsible for the development of psoriatic lesions. Abnormal proliferation of keratinocytes is thought to be related to defective epidermal cell surface receptors and altered intracellular signaling. The other choices do not describe increased numbers of otherwise normal epidermal cells. Diagnosis: Psoriasis, hyperplasia

A 30-year-old woman presents with a 2-month history of fatigue, mild fever, and an erythematous scaling rash. She also notes joint pain and swelling, primarily involving the small bones of her fingers. Physical examination reveals erythematous plaques with adherent silvery scales that induce punctate bleeding points when removed. Biopsy of lesional skin reveals markedly increased thickness of the epidermis (shown in the image). Which of the following terms best describes this adaptation to chronic injury in this patient with psoriasis? (A) Atrophy (B) Dysplasia (C) Hyperplasia (D) Hypertrophy (E) Metaplasia

*The answer is C.* The patient's biopsy shows normal skeletal muscle fibers on the left side of the image and a cluster of smaller fibers on the right side of the image. This pathologic finding is consistent with denervation muscle atrophy (reduced cell size and function). When a skeletal muscle fiber becomes separated from contact with its motor neuron, it invariably atrophies due to the progressive loss of myofibrils. On cross section, atrophic fibers have a characteristic angular configuration, seemingly compressed by the surrounding normal muscle fibers. If a muscle fiber is not reinnervated, atrophy proceeds to complete loss of myofibrils. In the end stage, muscle fibers disappear and are replaced by adipose tissue. None of the other choices describe histopathologic features of denervation muscle atrophy.

A 30-year-old woman suffers weakness of her right leg following an automobile accident, and a muscle biopsy is obtained (shown in the image). Compared to normal muscle fibers on the left, abnormal muscle fibers on the right side of this biopsy show which of the following histopathologic changes? (A) Abnormal pattern of myocyte differentiation (B) Accumulation of glycogen within myofibers (C) Decrease in the size of myocytes (D) Increased numbers of otherwise normal myocytes (E) Transformation of one differentiated cell type to another

*The answer is C.* Coagulative necrosis refers to light microscopic alterations in dying cells. When stained with the usual combination of hematoxylin and eosin, the cytoplasm of a necrotic cell is eosinophilic. The nucleus displays an initial clumping of chromatin followed by its redistribution along the nuclear membrane. In pyknosis, the nucleus becomes smaller and stains deeply basophilic as chromatin clumping continues. Karyorrhexis (choice B) and karyolysis (choice A) represent further steps in the fragmentation and dissolution of the nucleus. These steps are not evident in the necrotic cells shown in this Pap smear. Diagnosis: Cervical intraepithelial neoplasia, pyknosis

A 31-year-old woman complains of increased vaginal discharge of 1-month duration. A cervical Pap smear is shown in the image. Superficial epithelial cells are identified with arrows. When compared to cells from the deeper intermediate layer (top), the nuclei of these superficial cells exhibit which of the following cytologic features? (A) Karyolysis (B) Karyorrhexis (C) Pyknosis (D) Segmentation (E) Viral inclusion bodies

*The answer is C.* Inflammation from reflux of gastric acid has resulted in replacement of normal esophageal squamous epithelium by intestinal-type columnar epithelium with goblet cells. Such conversion of one adult cell type to another cell type is called metaplasia. The cells are not significantly increased in size (hypertrophic). The lamina propria has some inflammatory cells, but is not atrophic. Goblet cells are not normal constituents of the esophageal mucosa.

A 32-year-old man experiences "heartburn" and gastric reflux after eating a large meal. After many months of symptoms, he undergoes upper gastrointestinal endoscopy, and a biopsy specimen of the esophageal epithelium is obtained. Which of the following pathologic changes, seen in the figure, has most likely occurred? (A) Squamous metaplasia (B) Mucosal hypertrophy (C) Columnar-intestinal metaplasia (D) Atrophy of lamina propria (E) Goblet cell hyperplasia

*The answer is D.* Atrophy of an organ may be caused by interruption of key trophic signals. Postpartum infarction of the anterior pituitary in this patient resulted in decreased production of adrenocorticotropic hormone (ACTH, also termed corticotropin). Lack of corticotropin results in atrophy of the adrenal cortex, which leads to adrenal insufficiency. Symptoms of acute adrenal insufficiency (Addisonian crisis) include hypotension and shock, as well as weakness, vomiting, abdominal pain, and lethargy. The other choices are unlikely causes of postpartum adrenal insufficiency. Diagnosis: Sheehan syndrome, adrenal insufficiency

A 32-year-old woman develops an Addisonian crisis (acute adrenal insufficiency) 3 months after suffering massive hemorrhage during the delivery of her baby. A CT scan of the abdomen shows small adrenal glands. Which of the following mechanisms of disease best accounts for adrenal atrophy in this patient? (A) Chronic inflammation (B) Chronic ischemia (C) Hemorrhagic necrosis (D) Lack of trophic signals (E) Tuberculosis

*The answer is C.* Infants of diabetic mothers show a 5% to 10% incidence of major developmental abnormalities, including anomalies of the heart and great vessels and neural tube defects. The frequency of these lesions relates to the control of maternal diabetes during early gestation. During fetal development, the islet cells of the pancreas have proliferative capacity and respond to increased demand for insulin by undergoing physiologic hyperplasia. Fetuses exposed to hyperglycemia in utero may develop hyperplasia of the pancreatic β cells, which may secrete insulin autonomously and cause hypoglycemia at birth. Metaplasia (choice D) is defined as the conversion of one differentiated cell pathway to another. Diagnosis: Diabetes mellitus

A 32-year-old woman with poorly controlled diabetes mellitus delivers a healthy boy at 38 weeks of gestation. As a result of maternal hyperglycemia during pregnancy, pancreatic islets in the neonate would be expected to show which of the following morphologic responses to injury? (A) Atrophy (B) Dysplasia (C) Hyperplasia (D) Metaplasia (E) Necrosis

*The answer is C.* Cell death occurs with loss of the cell nucleus, and tubular cells become necrotic. All other cellular morphologic changes listed represent forms of reversible cellular injury. The plasma membrane and intracellular organelles retain some function unless severe damage causes loss of membrane integrity.

A 33-year-old woman has had increasing lethargy and decreased urine output for the past week. Laboratory studies show her serum creatinine is 4.3 mg/dL and urea nitrogen 40 mg/dL. A renal biopsy is performed, and the specimen is examined using electron microscopy. Which of the following morphologic cellular changes most likely suggests a diagnosis of acute tubular necrosis? A Chromatin clumping B Mitochondrial swelling C Nuclear fragmentation D Plasma membrane blebs E Ribosomal disaggregation

*The answer is C.* Loss of the nucleus results in cell death. All other cellular morphologic changes listed represent reversible cellular injury. The plasma membrane and intracellular organelles remain functional unless severe damage causes loss of membrane integrity.

A 33-year-old woman has had increasing lethargy and decreased urine output for the past week. Laboratory studies show serum creatinine level of 4.3 mg/dL and urea nitrogen level of 40 mg/dL. A renal biopsy is performed, and the specimen is examined using electron microscopy. Which of the following morphologic changes most likely suggests a diagnosis of acute tubular necrosis? (A) Mitochondrial swelling (B) Plasma membrane blebs (C) Karyolysis (D) Ribosomal disaggregation

*The answer is A.* Inflammation from reflux of gastric acid has resulted in replacement of normal esophageal squamous epithelium by intestinal-type columnar epithelium with goblet cells. Such conversion of one adult cell type to another cell type is called metaplasia, and it occurs when stimuli reprogram stem cells. Goblet cells are not normal constituents of the esophageal mucosa, and they are a minor part of this metaplastic process. The lamina propria has some inflammatory cells, but it does not atrophy. The squamous epithelium does not become dysplastic from acid reflux, but the columnar metaplasia may progress to dysplasia, not seen here, if the abnormal stimuli continue. These cells are not significantly increased in size (hypertrophic).

A 34-year-old obese woman has experienced heartburn from gastric reflux for the past 5 years after eating large meals. She undergoes upper gastrointestinal endoscopy, and a biopsy specimen of the distal esophagus is obtained. Which of the following microscopic changes, seen in the figure, has most likely occurred? A Columnar metaplasia B Goblet cell hyperplasia C Lamina propria atrophy D Squamous dysplasia E Mucosal hypertrophy

*The answer is B.* These paired images, obtained at the same magnification, contrast normal epidermis (image on left) with epidermal hyperplasia (image on right) in a patient with psoriasis. The epidermis in the patient with psoriasis is thickened, owing to an increase in the number of squamous epithelial cells. Psoriasis is a chronic disease of the dermis and epidermis that is characterized by persistent epidermal hyperplasia. It is frequently a familial disorder that features large, erythematous, scaly plaques, commonly on the dorsal extensor cutaneous surfaces of the body. Abnormal pattern of cellular maturation (choice A) is a description of dysplasia, which is not evident in this patient's biopsy. None of the other choices describe increased numbers of normal squamous epithelial cells in the epidermis.

A 34-year-old woman presents with a 3-month history of fatigue, mild fever, and skin rash. Physical examination reveals erythematous (red) plaques on the trunk and neck with silvery scales that induce bleeding when removed. Biopsy of lesional skin is obtained. Compared to normal skin (image on left), the patient's biopsy (image on right) shows which of the following histopathologic changes? (A) Abnormal pattern of cellular maturation (B) Increased numbers of otherwise normal cells (C) Invasion of neoplastic cells through the basement membrane (D) Transformation of one differentiated cell type to another (E) Ulceration and necrosis of the epithelium

*The answer is E.* The focal, chalky white deposits are areas of fat necrosis resulting from the release of pancreatic lipases in a patient with acute pancreatitis. Viral hepatitis does not cause necrosis in other organs, and hepatocyte necrosis from viral infections occurs mainly by means of apoptosis. Intestinal infarction is a form of coagulative necrosis. Tuberculosis produces caseous necrosis. Gangrenous necrosis is mainly coagulative necrosis, but occurs over an extensive area.

A 38-year-old woman experienced severe abdominal pain with hypotension and shock that led to her death within 36 hours after the onset of the pain. From the gross appearance of the mesentery, seen in the figure, which of the following events has most likely occurred? (A) Hepatitis B virus infection (B) Small intestinal infarction (C) Tuberculous lymphadenitis (D) Gangrenous cholecystitis (E) Acute pancreatitis

*The answer is A.* The many focal, chalky white deposits in the mesentery, composed mainly of adipocytes, are areas of fat necrosis. The deposits result from the release of pancreatic enzymes such as lipases in a patient with acute pancreatitis. Gangrenous necrosis is mainly coagulative necrosis, but occurs over an extensive area of tissues. Viral hepatitis does not cause cell necrosis in organs other than liver, and hepatocyte necrosis from viral infections occurs mainly by means of apoptosis. Intestinal infarction is a form of coagulative necrosis. Infection with tuberculosis leads to caseous necrosis.

A 38-year-old woman has experienced severe abdominal pain over the past day. On examination she is hypotensive and in shock. Laboratory studies show elevated serum lipase. From the representative gross appearance of the mesentery shown in the figure, which of the following events has most likely occurred? A Acute pancreatitis B Gangrenous cholecystitis C Hepatitis B virus infection D Small intestinal infarction E Tuberculous lymphadenitis

*The answer is C.* Fatty necrosis is a change in adipose tissue due to trauma or the release of enzymes from adjacent organs (e.g., the pancreas). . The trauma or enzymatic action causes a breakdown of lipid and a release of fatty acids, which combine with calcium to form chalky deposits.

A 40-year-old woman has the sudden onset of severe abdominal pain. On physical examination she has diffuse tenderness in all abdominal quadrants, with marked guarding and muscular rigidity. She has laboratory findings that include serum AST of 43 U/L, ALT of 30 U/L, LDH 630 U/L, and lipase 415 U/L. An abdominal CT scan reveals peritoneal fluid collections and decreased attenuation along with enlargement of the pancreas. Which of the following cellular changes is most likely to accompany these findings? A Coagulative necrosis B Dry gangrene C Fat necrosis D Apoptosis E Liquefactive necrosis

*The answer is A.* Fragmentation of DNA is a hallmark of cells undergoing both necrosis and apoptosis, but apoptotic cells can be detected by demonstrating nucleosomal "laddering." This pattern of DNA degradation is characteristic of apoptotic cell death. It results from the cleavage of chromosomal DNA at nucleosomes by endonucleases. Since nucleosomes are regularly spaced along the genome, a pattern of regular bands can be seen when fragments of cellular DNA are separated by electrophoresis. The other choices are associated with cell injury, but they do not serve as distinctive markers of programmed cell death. Diagnosis: Viral hepatitis

A 42-year-old man undergoes liver biopsy for evaluation of the grade and stage of his hepatitis C virus infection. The biopsy reveals swollen (ballooned) hepatocytes and moderate lobular inflammatory activity (shown in the image). The arrow identifies an acidophilic (Councilman) body. Which of the following biochemical changes characterizes the formation of acidophilic bodies in the patient described (A) Fragmentation of DNA (B) Loss of tumor suppressor protein p53 (C) Mitochondrial swelling (D) Synthesis of arachidonic acid (E) Triglyceride accumulation

*The answer is B.* Apoptosis is a programmed pathway of cell death that is triggered by a variety of extracellular and intracellular signals. It is often a self-defense mechanism, destroying cells that have been infected with pathogens or those in which genomic alterations have occurred. After staining with hematoxylin and eosin, apoptotic cells are visible under the light microscope as acidophilic (Councilman) bodies. These deeply eosinophilic structures represent membrane-bound cellular remnants that are extruded into the hepatic sinusoids. The other choices do not appear as acidophilic bodies. Diagnosis: Viral hepatitis

A 42-year-old man undergoes liver biopsy for evaluation of the grade and stage of his hepatitis C virus infection. The biopsy reveals swollen (ballooned) hepatocytes and moderate lobular inflammatory activity (shown in the image). The arrow identifies an acidophilic (Councilman) body. Which of the following cellular processes best accounts for the presence of scattered acidophilic bodies in this liver biopsy? (A) Aggregation of intermediate filament proteins (B) Apoptotic cell death (C) Coagulative necrosis (D) Collagen deposition (E) Intracellular viral inclusions

*The answer is E.* Metaplasia is the substitution of one tissue normally found at a site for another. The esophageal stratified squamous epithelium undergoes metaplasia in response to the ongoing inflammation from reflux of gastric contents. This is common in the lower esophagus with gastroesophageal reflux disease (GERD).

A 43-year-old man has complained of mild burning substernal pain following meals for the past 3 years. Upper GI endoscopy is performed and biopsies are taken of an erythematous area of the lower esophageal mucosa 3 cm above the gastroesophageal junction. There is no mass lesion, no ulceration, and no hemorrhage noted. The biopsies show the presence of columnar epithelium with goblet cells. Which of the following mucosal alterations is most likely represented by these findings? A Dysplasia B Hyperplasia C Carcinoma D Ischemia E Metaplasia

*The answer is B.* The mitochondrial membrane is a key regulator of apoptosis. When mitochondrial pores open, cytochrome c leaks out and activates Apaf-1, which converts procaspase-9 to caspase-9, resulting in the activation of downstream caspases (cysteine proteases). These effector caspases cleave target proteins, including endonucleases nuclear proteins, and cytoskeletal proteins to mediate the varied morphological and biochemical changes that accompany apoptosis. Reactive oxygen species (related to choices A, D, and E) are triggers of apoptosis, but they do not mediate programmed cell death. Diagnosis: Apoptosis, squamous cell carcinoma of skin

A 43-year-old man presents with a scaly, erythematous lesion on the dorsal surface of his left hand. A skin biopsy reveals atypical keratinocytes filling the entire thickness of the epidermis (shown in the image). The arrows point to apoptotic bodies. Which of the following proteins plays the most important role in mediating programmed cell death in this patient's skin cancer? (A) Catalase (B) Cytochrome c (C) Cytokeratins (D) Myeloperoxidase (E) Superoxide dismutase

*The answer is C.* The hypotension leads to diminished tissue perfusion with ischemic injury. Nuclear chromatin clumping is reversible, but nuclear pyknosis is not.

A 45-year-old man has a traumatic injury to his forearm and incurs extensive blood loss. On physical examination in the emergency department his blood pressure is 70/30 mm Hg. Which of the following cellular changes is most likely to represent irreversible cellular injury as a result of this injury? A Epithelial dysplasia B Cytoplasmic fatty metamorphosis C Nuclear pyknosis D Atrophy E Anaerobic glycolysis

*The answer is A.* The appearance of lipid vacuoles in many of the hepatocytes is characteristic of fatty change (steatosis) of the liver. Abnormalities in lipoprotein metabolism can lead to steatosis. Alcohol is a hepatotoxin acting via increased acetaldehyde accumulation that promotes hepatic steatosis. Decreased serum albumin levels and increased prothrombin time suggest alcohol-induced hepatocyte damage. Aspirin has a significant effect on platelet function, but not on hepatocytes. Substance abuse with heroin produces few organ-specific pathologic findings. Exercise has little direct effect on hepatic function. Smoking directly damages lung tissue, but has no direct effect on the liver.

A 46-year-old man has noted increasing abdominal size for the past 6 years. On physical examination his liver span is increased to 18 cm. An abdominal CT scan shows an enlarged liver with diffusely decreased attenuation. Laboratory findings include increased total serum cholesterol and triglyceride levels, increased prothrombin time, and a decreased serum albumin concentration. The representative microscopic appearance of his liver is shown in the figure. Which of the following activities most likely led to these findings? A Drinking beer B Ingesting aspirin C Injecting heroin D Playing basketball E Smoking cigarettes

*The answer is B.* The irritant effect, such as the various chemicals in cigarette smoke, leads to replacement of the normal epithelium with another (such as squamous epithelium replacing respiratory epithelium). This metaplastic process is the first step that could lead to dysplasia and then to neoplasia

A 46-year-old man has smoked 2 packs of cigarettes per day for the past 30 years. He has had a chronic cough for the past 3 years, worsening over the last 2 weeks. A suspicious left pulmonary parenchymal lesion is seen on a chest x-ray. He has a bronchoscopy performed. A biopsy of a segmental bronchus shows squamous metaplasia. Which of the following is the most appropriate interpretation of this finding? A Ischemic tissue damage B Irritant effect C Early stage of cancer D Viral infection E Congenital anomaly

*The answer is E.* Metaplasia is almost invariably a response to persistent injury and can be thought of as an adaptive mechanism. Prolonged exposure of the bronchi to tobacco smoke leads to squamous metaplasia of the bronchial epithelium. Unlike malignancy (choice B) and necrosis with scarring (choice C), metaplasia is usually fully reversible. If the source of injury in this patient is removed (the patient stops smoking), then the metaplastic epithelium will eventually return to normal. Diagnosis: Chronic bronchitis, metaplasia

A 47-year-old man with a history of heavy smoking complains of chronic cough. A "coin lesion" is discovered in his right upper lobe on chest X-ray. Bronchoscopy and biopsy fail to identify a mass, but the bronchial mucosa displays squamous metaplasia. What is the most likely outcome of this morphologic adaptation if the patient stops smoking? (A) Atrophy (B) Malignant transformation (C) Necrosis and scarring (D) Persistence throughout life (E) Reversion to normal

*The answer is D.* Decreased tissue perfusion from hypotensive shock leads to hypoxemia and depletion of ATP when cell metabolism shifts from aerobic to anaerobic glycolysis. This shift causes depletion of glycogen stores and increased production and accumulation of lactic acid, reducing intracellular pH. Creatinine would increase with reduced renal function from decreased renal perfusion, but this would not explain the changes in other tissues. An increased glucose level would be indicative of poorly controlled diabetes mellitus, not decreased perfusion. Carbon dioxide is likely to be cleared via normal lungs, which are still sufficiently perfused by a failing heart. An increase in troponin I suggests irreversible myocardial injury.

A 47-year-old woman has poorly controlled diabetes mellitus and develops coronary artery disease. She now has decreasing cardiac output with blood pressure of 80/40 mm Hg and ejection fraction of 18%. An increase in which of the following substances in her blood is most indicative of reversible cell injury from decreased systemic arterial perfusion of multiple organs and tissues? A Carbon dioxide B Creatinine C Glucose D Lactic acid E Troponin I

*The answer is B.* The toxic effects of the alcohol culminate in large lipid droplet accumulation within hepatocytes. Over time, more hepatocytes are affected and the liver is less able to compensate. At this point, the man retains sufficient hepatocyte function.

A 48-year-old man has a history of chronic alcohol abuse. He is still able to perform work at his job. He has had no major illnesses. On physical examination, there are no significant findings. Laboratory studies are within normal limits. Which of the following microscopic findings in his liver is most likely to be present? A Cholestasis B Fatty change C Hemochromatosis D Hypertrophy of smooth endoplasmic reticulum E Coagulative necrosis

*The answer is E.* The induction of individual cell death occurs in the process of apoptosis. The drug effect is targeted primarily at the neoplastic cells, not normal cells.

A 48-year-old woman has a malignant lymphoma involving lymph nodes in the para-aortic region. She is treated with a chemotherapeutic agent which results in the loss of individual neoplastic cells through fragmentation of individual cell nuclei and cytoplasm. Over the next 2 months, the lymphoma decreases in size, as documented on abdominal CT scans. By which of the following mechanisms has her neoplasm primarily responded to therapy? a) Coagulative necrosis b) Mitochondrial poisoning c) Phagocytosis d) Acute inflammation e) Apoptosis

*The answer is C.* Myositis ossificans is a disease characterized by formation of bony trabeculae within striated muscle. It represents a form of osseous metaplasia (i.e., replacement of one differentiated tissue with another type of normal differentiated tissue). Although dystrophic calcification (choice E) frequently occurs at sites of prior injury, it does not lead to the formation of bone trabeculae. Diagnosis: Myositis ossifi cans, metaplasia

A 5-year-old boy suffers blunt trauma to the leg in an automobile accident. Six months later, bone trabeculae have formed within the striated skeletal muscle at the site of tissue injury. This pathologic condition is an example of which of the following morphologic adaptations to injury? (A) Atrophy (B) Dysplasia (C) Metaplasia (D) Metastatic calcification (E) Dystrophic calcification

*The answer is C.* Saponification of fat derived from peripancreatic fat cells exposed to pancreatic enzymes is a typical feature of fat necrosis. Lipase, released from pancreatic acinar cells during an attack of acute pancreatitis, hydrolyzes fat into fatty acids and glycerol. Free fatty acids bind with calcium to form soaps, which is a process known as saponification. Entry of calcium ions into the injured tissue reduces the level of calcium in blood. Hypocalcemia is, therefore, a typical finding in patients who had a recent bout of acute pancreatitis. Patients with acute pancreatitis experience sudden-onset abdominal pain, distention, and vomiting. The other choices are not typically seen in peripancreatic tissue following acute pancreatitis, although liquefactive necrosis (choice E) may be observed. Diagnosis: Acute pancreatitis

A 50-year-old chronic alcoholic presents to the emergency room with 12 hours of severe abdominal pain. The pain radiates to the back and is associated with an urge to vomit. Physical examination discloses exquisite abdominal tenderness. Laboratory studies show elevated serum amylase. Which of the following morphologic changes would be expected in the peripancreatic tissue of this patient? (A) Coagulative necrosis (B) Caseous necrosis (C) Fat necrosis (D) Fibrinoid necrosis (E) Liquefactive necrosis

*The answer is A.* The patient's bladder biopsy shows squamous metaplasia of the urothelium. In this patient, parasitic infestation of the bladder caused the normal transitional cell epithelium to be replaced by a protective, stratified squamous epithelium. Squamous cell carcinoma of the bladder can develop in foci of squamous metaplasia, and a high incidence of bladder cancer is found in countries with endemic schistosomiasis. The other choices do not reflect squamous metaplasia of the bladder mucosa.

A 50-year-old man from Egypt presents with blood in his urine (hematuria). The patient's past medical history is significant for a parasitic infestation of his bladder (schistosomiasis). A biopsy of the bladder wall is obtained. How does the microscopic appearance of the patient's bladder (image on right) differ from that of normal bladder epithelium (image on left)? (A) The cell type of the epithelium has been changed. (B) The epithelium shows new uncontrolled growth. (C) The lamina propria of the mucosa is absent. (D) There is an infectious process within the epithelium. (E) There is erosion of the surface epithelium.

*The answer is A.* The figure shows deep eosinophilic staining, loss of myocardial fiber nuclei, and loss of cell structure consistent with an early ischemic injury, resulting in coagulative necrosis. Myocardial ischemia and infarction are typically caused by loss of coronary arterial blood flow. An immunological process may produce focal myocardial injury. Blunt trauma produces hemorrhage. Lack of protein leads to a catabolic state with gradual decrease in cell size, but it does not cause ischemic changes. Viral infection could cause focal necrosis of the myocardium, but this is usually accompanied by an inflammatory infiltrate consisting of lymphocytes and macrophages.

A 50-year-old man has experienced an episode of chest pain for 6 hours. A representative histologic section of his left ventricular myocardium is shown in the figure. There is no hemorrhage or inflammation. Which of the following conditions most likely produced these myocardial changes? A Arterial thrombosis B Autoimmunity C Blunt chest trauma D Protein-deficient diet E Viral infection

*The answer is A.* Cellular swelling is reversible.

A 50-year-old woman comes to the emergency department with sudden onset left-sided calf pain starting 20 minutes ago. Her past medical history is notable for diabetes, hypertension, and chronic atrial fibrillation controlled on an anti-arrhythmic. Examination of the leg reveals a pale left calf with diminished dorsalis pedis and posterior tibial pulses. Prompt intervention relieves the patient's symptoms. Which of the following, if seen microscopically, would indicate that her calf myocytes are more likely to recover without permanent damage? A. Cellular swelling B. Mitochondrial dense bodies C. Nuclear pyknosis D. Plasma membrane rupture E. Fragmented cells and debris

*The answer is C.* Such toxic oxygen radicals are released from neutrophils when blood flow is restored following ischemia. This is a reperfusion injury. Overall, there is likely to be more good than harm to restoration of blood flow.

A 50-year-old woman with a history of unstable angina suffers an acute myocardial infarction. Thrombolytic therapy with tissue plasminogen activator (tPA) is administered to restore coronary blood flow. In spite of this therapy, the degree of myocardial injury may increase because of which of the following cellular abnormalities? A Cytoskeletal intermediate filament loss B Decreased intracellular pH from anaerobic glycolysis C Increased free radical formation D Mitochondrial swelling E Nuclear chromatin clumping F Reduced protein synthesis

*The answer is D.* Hypertrophy is a response to trophic signals or increased functional demand and is commonly a normal process. For example, if one kidney is rendered inoperative because of vascular occlusion, the contralateral kidney hypertrophies to accommodate increased demand. The molecular basis of hypertrophy reflects increased expression of growth-promoting genes (protooncogenes) such as myc, fos, and ras. Hyperplasia (choice C) of renal tubular cells may occur, but enlargement of the kidney in this patient is best referred to as hypertrophy (i.e., increased organ size and function). Diagnosis: Hypertrophy

A 52-year-old woman loses her right kidney following an automobile accident. A CT scan of the abdomen 2 years later shows marked enlargement of the left kidney. The renal enlargement is an example of which of the following adaptations? (A) Atrophy (B) Dysplasia (C) Hyperplasia (D) Hypertrophy (E) Metaplasia

*The answer is C.* Chromatin clumping is reversible, but dissolution of the entire nucleus is not, and when the nucleus is lost, the cell will die. An acute coronary syndrome with myocardial infarction is described here.

A 53-year-old man has experienced severe chest pain for the past 6 hours. On physical examination he is afebrile, but has tachycardia. Laboratory studies show a serum troponin I of 10 ng/mL. A coronary angiogram is performed emergently and reveals >90% occlusion of the anterior interventricular (left anterior descending) artery. In this setting, an irreversible injury to myocardial fibers will have occurred when which of the following cellular changes occurs? A Glycogen stores are depleted B Cytoplasmic sodium increases C Nuclei undergo karyorrhexis D Intracellular pH diminishes E Blebs form on cell membranes

*The answer is F.* The pressure load on the left ventricle results in an increase in myofilaments in the existing myofibers, so they enlarge. The result of continued stress from hypertension is eventual heart failure with decreased contractility. Apoptosis would lead to loss of cells and diminished size. Dysplasia is not a diagnosis made for the heart. Hemosiderin deposition in the heart is a pathologic process resulting from increased iron stores in the body. Though hyperplasia from proliferation of myofibroblasts is possible, this does not contribute significantly to cardiac size. Metaplasia of muscle does not occur, although loss of muscle occurs with aging and ischemia as myofibers are replaced by fibrous tissue.

A 53-year-old woman with no prior illnesses has a routine checkup by her physician. On examination she has a blood pressure of 150/95 mm Hg. If her hypertension remains untreated for years, which of the following cellular alterations would most likely be seen in her myocardium? A Apoptosis B Dysplasia C Fatty change D Hemosiderosis E Hyperplasia F Hypertrophy G Metaplasia

*The answer is A.* If the existing cell damage is not great after myocardial infarction, the restoration of blood flow can help prevent further damage. The reperfusion of damaged cells results in generation of oxygen-derived free radicals, however, causing a reperfusion injury. The elevation in the CK level is indicative of myocardial cell necrosis because this intracellular enzyme does not leak in large quantities from intact cells. Myocardial fibers do not regenerate to a significant degree, and atrophic fibers would have less enzyme to release. t-PA does not produce a chemical injury; it induces thrombolysis to restore blood flow in blocked coronary arteries.

A 54-year-old man experienced onset of severe substernal chest pain over 3 hours. An ECG showed changes consistent with an acute myocardial infarction. After thrombolytic therapy with tissue plasminogen activator (t-PA), his serum creatine kinase (CK) level increased. Which of the following events most likely occurred after t-PA therapy? □ (A) Reperfusion injury □ (B) Cellular regeneration □ (C) Chemical injury □ (D) Increased synthesis of CK □ (E) Myofiber atrophy

*The answer is A.* Irreversible cell injury is associated with loss of membrane integrity. This allows intracellular enzymes such as AST and ALT to leak into the serum. All other morphologic changes listed are associated with reversible cell injury, in which the cell membrane remains intact and the cells do not die.

An 11-year-old girl becomes infected with hepatitis A and experiences mild nausea for 1 week. On physical examination, she has minimal right upper quadrant tenderness and scleral icterus. Laboratory findings include a serum AST of 68 U/L, ALT of 75 U/L, and total bilirubin of 5.1 mg/dL. Her laboratory findings most likely result from which of the following changes in her hepatocytes? A Cell membrane defects B Lysosomal autophagy C Mitochondrial swelling D Nuclear chromatin clumping E Ribosomal dispersion

*The answer is E.* If existing cell damage is not great after myocardial infarction, the restoration of blood flow can help prevent further cellular damage. However, the reperfusion of damaged cells results in generation of oxygen-derived free radicals, causing a reperfusion injury. The elevation in the CK level is indicative of myocardial cell necrosis, because this intracellular enzyme does not leak in large quantities from intact myocardial cells. Myocardial fibers do not regenerate to a significant degree, and atrophic fibers would have less CK to release. t-PA does not produce a toxic chemical injury; it induces thrombolysis to restore blood flow in occluded coronary arteries.

A 54-year-old man experienced severe substernal chest pain for 3 hours. An ECG showed changes consistent with an acute myocardial infarction. After thrombolytic therapy with tissue plasminogen activator (t-PA), his serum creatine kinase (CK) level increased. Which of the following tissue events most likely occurred in the myocardium after t-PA therapy? A Cellular regeneration B Drug toxicity C Increased synthesis of CK D Myofiber atrophy E Reperfusion injury

*The answer is D.* The loss of normal functioning respiratory epithelium with loss of mucociliary protection means that the normal innate barrier to infection has been weakened.

A 56-year-old woman has smoked 2 packs of cigarettes per day for the past 35 years. She has had a chronic cough for the past 8 years, but recently has noted increased sputum production. On physical examination she has a few crackles auscultated best over the lung bases. Bronchoscopy with biopsy is performed. The biopsy reveals bronchial epithelium with squamous metaplasia. Which of the following statements represents the best interpretation of these findings? A Physiologic process of aging B Irreversible, even if she stops smoking C Metastases to the lung D Risk for infection E Thromboembolism with infarction

*The answer is A.* Interference with blood supply to tissues is known as ischemia. Total ischemia results in cell death. Partial ischemia occurs after incomplete occlusion of a blood vessel or in areas of inadequate collateral circulation. This results in a chronically reduced oxygen supply, a condition often compatible with continued cell viability. Under such circumstances, cell atrophy is common. For example, it is frequently seen around the inadequately perfused margins of infarcts in the heart, brain, and kidneys. None of the other choices describe decreased organ size and function. Diagnosis: Renal artery stenosis

A 56-year-old woman with a history of hyperlipidemia and hypertension develops progressive, right renal artery stenosis. Over time, this patient's right kidney is likely to demonstrate which of the following morphologic adaptations to partial ischemia? (A) Atrophy (B) Dysplasia (C) Hyperplasia (D) Hypertrophy (E) Neoplasia

*The answer is D.* Fibrinoid necrosis is an alteration of injured blood vessels, in which the insudation and accumulation of plasma proteins cause the wall to stain intensely with eosin. The other choices are not typically associated directly with vascular injury. Diagnosis: Malignant hypertension, fi brinoid necrosis

A 58-year-old man presents with symptoms of acute renal failure. His blood pressure is 220/130 mm Hg (malignant hypertension). While in the emergency room, the patient suffers a stroke and expires. Microscopic examination of the kidney at autopsy is shown in the image. Which of the following morphologic changes accounts for the red material in the wall of the artery? (A) Apoptosis (B) Caseous necrosis (C) Fat necrosis (D) Fibrinoid necrosis (E) Liquefactive necrosis

*The answer is E.* When the rate of dissolution of the necrotic cells is faster than the rate of repair, the resulting morphologic appearance is termed liquefactive necrosis. The polymorphonuclear leukocytes of the acute inflammatory reaction are endowed with potent hydrolases that are capable of digesting dead cells. A sharply localized collection of these acute inflammatory cells in response to a bacterial infection produces rapid death and dissolution of tissue. The result is often an abscess defined as a cavity formed by liquefactive necrosis in a solid tissue. Caseous necrosis (choice A) is seen in necrotizing granulomas. In coagulative necrosis (choice B), the outline of the cell is retained. Fat (choice C) is not present in the lung parenchyma. Fibrinoid necrosis (choice D) is seen in patients with necrotizing vasculitis. Diagnosis: Pulmonary abscess, liquefactive necrosis

A 59-year-old female alcoholic is brought to the emergency room with a fever (38.7°C/103°F) and foul-smelling breath. The patient subsequently develops acute bronchopneumonia and dies of respiratory insufficiency. A pulmonary abscess is identified at autopsy (shown in the image). Histologic examination of the wall of this lesion would most likely demonstrate which of the following pathologic changes? (A) Caseous necrosis (B) Coagulative necrosis (C) Fat necrosis (D) Fibrinoid necrosis (E) Liquefactive necrosis

*The answer is C.* Renal cell carcinomas often secrete erythropoietin. This hormone stimulates the growth of erythrocyte precursors in the bone marrow by inhibiting programmed cell death. Increased hematocrit in this patient is the result of bone marrow hyperplasia affecting the erythroid lineage. The other choices do not represent physiologic responses to erythropoietin. Diagnosis: Renal cell carcinoma, hyperplasia

A 59-year-old woman smoker complains of intermittent blood in her urine. Urinalysis confi rms 4+ hematuria. A CBC reveals increased red cell mass (hematocrit). A CT scan demonstrates a 3-cm renal mass, and a CT-guided biopsy displays renal cell carcinoma. Which of the following cellular adaptations in the bone marrow best explains the increased hematocrit in this patient? (A) Atrophy (B) Dysplasia (C) Hyperplasia (D) Hypertrophy (E) Metaplasia

*The answer is E.* Metaplasia of transitional epithelium to glandular epithelium is seen in patients with chronic inflammation of the bladder (cystitis glandularis). Metaplasia is considered to be a protective mechanism, but it is not necessarily a harmless process. For example, squamous metaplasia in a bronchus may protect against injury produced by tobacco smoke, but it also impairs the production of mucus and ciliary clearance of debris. Furthermore, neoplastic transformation may occur in metaplastic epithelium. Lack of cytologic evidence for atypia and neoplasia rules out dysplasia (choice B). Diagnosis: Chronic cystitis, metaplasia

A 60-year-old man with chronic cystitis complains of urinary frequency and pelvic discomfort. Digital rectal examination is unremarkable. Biopsy of the bladder mucosa reveals foci of glandular epithelium and chronic inflammatory cells. No cytologic signs of atypia or malignancy are observed. Which of the following terms best describes the morphologic response to chronic injury in this patient? (A) Atrophy (B) Dysplasia (C) Hyperplasia (D) Hypertrophy (E) Metaplasia

*The answer is B.* Embolization of the thrombus led to blockage of a renal arterial branch, causing an acute renal infarction in this patient. An ischemic injury to most internal organs produces a pattern of cell death called coagulative necrosis. Note the faint outlines of renal tubules and glomerulus in the figure, but no cellular nuclei. Caseous necrosis can be seen in various forms of granulomatous inflammation, typified by tuberculosis. Fat necrosis is usually seen in pancreatic and breast tissue. Gangrenous necrosis is a form of coagulative necrosis that usually results from ischemia and affects limbs. Liquefactive necrosis occurs after ischemic injury to the brain and is the pattern seen with abscess formation.

A 63-year-old man has a 2-year history of worsening congestive heart failure. An echocardiogram shows mitral valve stenosis with left atrial dilation. A mural thrombus is present in the left atrium. One month later, he experiences left flank pain and notes hematuria. Laboratory testing shows an elevated serum AST. The representative microscopic appearance of the lesion is shown in the figure. Which of the following patterns of tissue necrosis is most likely to be present in this man? A Caseous B Coagulative C Fat D Gangrenous E Liquefactive

*The answer is C.* Hypertrophic cardiac myocytes have more cytoplasm and larger nuclei than normal cells. Although the elucidation of the cellular and molecular mechanisms underlying the hypertrophic response is still actively pursued, it is clear that the final steps include increases in mRNA, rRNA, and protein. Hypertrophy results from transcriptional regulation. Aneuploidy (choice A) is not a feature of myofiber hypertrophy. Water influx (choice E), which is typical of hydropic swelling in acute injury, is not a common feature of hypertrophy. Diagnosis: Hypertrophic heart disease, hypertrophy

A 64-year-old man with long-standing angina pectoris and arterial hypertension dies of spontaneous intracerebral hemorrhage. At autopsy, the heart appears globoid. The left ventricle measures 2.8 cm on cross section (shown in the image). This adaptation to chronic injury was mediated primarily by changes in the intracellular concentration of which of the following components? (A) DNA (B) Glycogen (C) Lipid (D) mRNA (E) Water

*The answer is B.* Cell swelling is reversible. The tPA provided reperfusion so that areas of brain that were ischemic but not infarcted could recover. However, waiting too long to restore circulation could increase free radical formation and reperfusion injury.

A 65-year-old woman has the sudden inability to move her right arm and to speak. MR angiography shows occlusion of a cerebral artery. She is given tissue plasminogen activator (tPA). Over the next week she regains some ability to move her arm and to speak. Which of the following cellular changes most likely subsided upon tPA therapy? a) Cell fragmentation b) Cell swelling c) Membrane disruption d) Nuclear karyorrhexis e) Nuclear pyknosis

*The answer is C.* The major adaptive responses of cells to sublethal injury are atrophy, hypertrophy, hyperplasia, metaplasia, dysplasia, and intracellular storage. Metaplasia is defined as the conversion of one differentiated cell pathway to another. In this case, the esophageal squamous epithelium is replaced by columnar epithelium as a result of chronic gastroesophageal reflux. The lesion is characterized histologically by intestine-like epithelium composed of goblet cells and cells similar to those of incompletely intestinalized gastric mucosa. Squamous metaplasia (choice E) occurs in the bronchial epithelium of smokers, among other examples. Choices A, B, and D are preneoplastic changes that are most often described in the uterine endometrium of postmenopausal women. Diagnosis: Barrett esophagus, metaplasia

A 68-year-old man with a history of gastroesophageal refl ux disease suffers a massive stroke and expires. The esophagus at autopsy is shown in the image. Histologic examination of the abnormal tissue shows intestine-like epithelium composed of goblet cells and surface cells similar to those of incompletely intestinalized gastric mucosa. There is no evidence of nuclear atypia. Which of the following terms best describes this morphologic response to persistent injury in the esophagus of this patient? (A) Atypical hyperplasia (B) Complex hyperplasia (C) Glandular metaplasia (D) Simple hyperplasia (E) Squamous metaplasia

*The answer is E.* The high lipid content of central nervous system tissues results in liquefactive necrosis as a consequence of ischemic injury, as in this case of stroke. Apoptosis affects single cells and typically is not grossly visible. Coagulative necrosis is the typical result of ischemia in most solid organs. Fat necrosis is seen in breast and pancreatic tissues. Karyolysis refers to fading away of cell nuclei in dead cells.

A 68-year-old woman suddenly lost consciousness and on awakening 1 hour later, she could not speak or move her right arm. Two months later, a head CT scan showed a large cystic area in the left parietal lobe. Which of the following pathologic processes has most likely occurred in her brain? A Apoptosis B Coagulative necrosis C Fat necrosis D Karyolysis E Liquefactive necrosis

*The answer is D.* The high lipid content of central nervous system tissues results in liquefactive necrosis as a consequence of ischemic injury, as in this case of a "stroke." Fat necrosis is seen in breast and pancreatic tissues. Coagulative necrosis is the typical result of ischemia in most solid organs. Apoptosis affects single cells and typically is not grossly visible. Karyolysis refers to fading away of cell nuclei in dead cells.

A 68-year-old woman suddenly lost consciousness; on awakening 1 hour later, she could not speak or move her right arm and leg. Two months later, a head CT scan showed a large cystic area in the left parietal lobe. Which of the following pathologic processes has most likely occurred in the brain? (A) Fat necrosis (B) Coagulative necrosis (C) Apoptosis (D) Liquefactive necrosis (E) Karyolysis

*The answer is D.* During lipid peroxidation, hydroxyl radicals remove a hydrogen atom from the unsaturated fatty acids of membrane phospholipids. The lipid radicals so formed react with molecular oxygen and form a lipid peroxide radical. A chain reaction is initiated. Lipid peroxides are unstable and break down into smaller molecules. The destruction of the unsaturated fatty acids of phospholipids results in a loss of membrane integrity. The other choices represent targets for reactive oxygen species, but protein cross-linking (choices B and E) does not lead to rapid loss of membrane integrity in patients with myocardial infarction.

A 68-year-old woman with a history of hyperlipidemia dies of cardiac arrhythmia following a massive heart attack. Peroxidation of which of the following molecules was primarily responsible for causing the loss of membrane integrity in cardiac myocytes in this patient? (A) Cholesterol (B) Glucose transport proteins (C) Glycosphingolipids (D) Phospholipids (E) Sodium-potassium ATPase

*The answer is D.* Nodular prostatic hyperplasia (also known as benign prostatic hyperplasia [BPH]) is a common condition in older men that results from proliferation of prostatic glands and stroma. The prostate becomes more sensitive to androgenic stimulation with age. This is an example of pathologic hyperplasia. Apoptosis results in a loss of, not an increase in, cells. Dysplasia refers to disordered epithelial cell growth and maturation. Fatty change in hepatocytes may produce hepatomegaly. Although BPH is often called "benign prostatic hypertrophy," this term is technically incorrect; it is the number of glands and stromal cells that is increased, rather than the size of existing cells. A change in the glandular epithelium to squamous epithelium would be an example of metaplasia.

A 69-year-old man has had difficulty with urination, including hesitancy and frequency, for the past 5 years. A digital rectal examination reveals that the prostate gland is palpably enlarged to about twice normal size. A transurethral resection of the prostate is performed, and the microscopic appearance of the prostate "chips" obtained is that of nodules of glands with intervening stroma. Which of the following pathologic processes has most likely occurred in the prostate? (A) Apoptosis (B) Dysplasia (C) Fatty change (D) Hyperplasia (E) Hypertrophy (F) Metaplasia

*The answer is E.* The esophagus is a 25-cm tube that passes through the superior and inferior mediastinum. It enters the abdominal cavity at vertebral level T10 and delivers food to the stomach for mechanical and enzymatic digestion. This autopsy specimen was obtained from a patient with a history of chronic gastroesophageal (acid) reflux. It shows evidence of both normal (upper) and abnormal (lower) esophageal mucosa. The normal mucosa that lines the proximal portion of this esophagus (on the left) exhibits a nonkeratinized stratified squamous epithelium. Rugae are noted in the cardia of the stomach (on the right). None of the other types of lining epithelium describe histologic features of the esophagus.

A 69-year-old man with a history of gastroesophageal reflux develops a pulmonary saddle embolus and expires. The patient's esophagus is examined at autopsy (shown in the image). Which of the following types of epithelium lines the proximal portion of this autopsy specimen? (A) Pseudostratified columnar with cilia and goblet cells (B) Pseudostratified columnar with goblet cells (C) Simple columnar with goblet cells (D) Stratified squamous, keratinized (E) Stratified squamous, nonkeratinized

*The answer is E.* Nuclear fragmentation (karyorrhexis and karyolysis) is a hallmark of coagulative necrosis. Choices A, B, and D are incorrect because they are features of both reversibly and irreversibly injured cells. Lymphocytes (choice C) are a hallmark of chronic inflammation. Diagnosis: Myocardial infarction

A 70-year-old man is hospitalized after suffering a mild stroke. While in the hospital, he suddenly develops crushing substernal chest pain. Analysis of serum proteins and ECG confirm a diagnosis of acute myocardial infarction. The patient subsequently develops an arrhythmia and expires. A cross section of the left ventricle at autopsy is shown in the image. Which of the following histologic features would provide definitive evidence of necrosis in the myocardium of the patient described? (A) Disaggregation of polyribosomes (B) Increased intracellular volume (C) Influx of lymphocytes (D) Mitochondrial swelling and calcification (E) Nuclear fragmentation

*The answer is B.* Ischemic necrosis of cardiac myocytes is the leading cause of death in the Western world. In brief, the interruption of blood supply to the heart decreases the delivery of O2 and glucose. Lack of O2 impairs mitochondrial electron transport, thereby decreasing ATP synthesis and facilitating the production of reactive oxygen species. Mitochondrial damage promotes the release of cytochrome c to the cytosol, and the cell dies. The morphologic appearance of the necrotic cell has traditionally been termed coagulative necrosis because of its similarity to the coagulation of proteins that occurs upon heating. Diagnosis: Myocardial infarction, coagulative necrosis

A 70-year-old man is hospitalized after suffering a mild stroke. While in the hospital, he suddenly develops crushing substernal chest pain. Analysis of serum proteins and ECG confirm a diagnosis of acute myocardial infarction. The patient subsequently develops an arrhythmia and expires. A cross section of the left ventricle at autopsy is shown in the image. Histologic examination of the affected heart muscle would demonstrate which of the following morphologic changes? (A) Caseous necrosis (B) Coagulative necrosis (C) Fat necrosis (D) Fibrinoid necrosis (E) Liquefactive necrosis

*The answer is C.* Prostatic hyperplasia is an example of pathologic hyperplasia in which prostatic cells become more sensitive to hormonal stimuli for growth. Both glands and stroma can increase in amount. Hyperplasia is an increase in the number of cells.

A 71-year-old man has difficulty with urination. His urinary retention leads to numerous trips to the restroom per day. On digital rectal examination is prostate is diffusely enlarged. Which of the following represents a pathologic change leading to this man's problem? A Dysplasia B Hypertrophy C Hyperplasia D Metaplasia E Neoplasia

*The answer is D.* Nodular prostatic hyperplasia (also known as benign prostatic hyperplasia [BPH]) is a common condition in older men that results from proliferation of both prostatic glands and stroma. The prostate becomes more sensitive to androgenic stimulation with age. This is an example of pathologic hyperplasia. Apoptosis results in a loss of, not an increase in, cells. Dysplasia refers to disordered epithelial cell growth and maturation. Fatty change in hepatocytes may produce hepatomegaly. Although BPH is often called "benign prostatic hypertrophy," this term is technically incorrect; it is the number of glands and stromal cells that is increased, rather than the size of existing cells. A change in the glandular epithelium to squamous epithelium around a prostatic infarct would be an example of metaplasia.

A 71-year-old man has had difficulty with urination, including hesitancy and increased frequency, for the past 5 years. A digital rectal examination reveals that his prostate gland is palpably enlarged to twice normal size. A transurethral resection of the prostate is performed, and the microscopic appearance of the prostate "chips" obtained is that of nodules of glands with intervening stroma. Which of the following pathologic processes has most likely occurred in his prostate? A Apoptosis B Dysplasia C Fatty change D Hyperplasia E Hypertrophy F Metaplasia

*The answer is B.* The valve is stenotic because of nodular deposits of calcium. The process is "dystrophic" because calcium deposition occurs in damaged tissues. The damage in this patient is a result of the wear and tear of aging. Amyloid deposition in the heart typically occurs within the myocardium and the vessels. The amount of lipofuscin increases within myocardial fibers (not valves) with aging. Hereditary hemochromatosis is a genetic defect in iron absorption that results in extensive myocardial iron deposition (hemosiderosis). Fatty change is uncommonly seen in myocardium, but infiltration of fat cells between myofibers can occur.

A 72-year-old man died suddenly from congestive heart failure. At autopsy, the heart weighed 580 g and showed marked left ventricular hypertrophy and minimal coronary arterial atherosclerosis. A serum chemistry panel ordered before death showed no abnormalities. Which of the following pathologic processes best accounts for the appearance of the aortic valve seen in the figure? (A) Amyloidosis (B) Dystrophic calcification (C) Lipofuscin deposition (D) Hemosiderosis (E) Fatty change

*The answer is C.* These paired images contrast normal bone marrow (image on left) with hyperplastic bone marrow (image on right) from a patient living at high altitude. As an adaptation to low atmospheric oxygen content, erythrocyte precursors in the patient's bone marrow undergo compensatory hyperplasia to increase the delivery of oxygen-carrying erythrocytes to the blood. Note that cellularity of the patient's bone marrow is increased and that fat content is decreased. None of the other choices describe increased numbers of hematopoietic cells in the bone marrow.

A 74-year-old man from a mountain region of the country develops respiratory insufficiency and expires. The patient's organs are examined at autopsy. Compared to a normal bone marrow (image on left), the patient's bone marrow (image on right) demonstrates which of the following reversible adaptive changes? (A) Atrophy (decreased cell size/function) (B) Dysplasia (abnormal maturation) (C) Hyperplasia (increased numbers of cells) (D) Hypertrophy (increased cell size/function) (E) Metaplasia (new differentiation pathway)

*The answer is B.* Clinically, atrophy is recognized as diminution in the size or function of an organ. It is often seen in areas of vascular insufficiency or chronic inflammation and may result from disuse. Atrophy may be thought of as an adaptive response to stress, in which the cell shuts down its differentiated functions. Reduction in the size of an organ may reflect reversible cell atrophy or may be caused by irreversible loss of cells. For example, atrophy of the brain in this patient with Alzheimer disease is secondary to extensive cell death, and the size of the organ cannot be restored. This patient's brain shows marked atrophy of the frontal lobe. The gyri are thinned, and sulci are widened. Anaplasia (choice A) represents lack of differentiated features in a neoplasm. Diagnosis: Alzheimer disease, atrophy

A 75-year-old woman with Alzheimer disease dies of congestive heart failure. The brain at autopsy is shown in the image. This patient's brain exemplifies which of the following responses to chronic injury? (A) Anaplasia (B) Atrophy (C) Dysplasia (D) Hyperplasia (E) Hypertrophy

*The answer is A.* Normal cells handle physiologic demands and maintain metabolic functions within narrow ranges, termed homeostasis. Under disease conditions with stress on cells, there is adaptation to a new steady state. In this case, the loss of renal function leads to a higher urea nitrogen level as well as retention of fluid. The diuretic induces loss of the excess fluid to yield a new steady state. The protein restriction reduces urea nitrogen excretion, which also leads to a new steady state. Both are adaptations. Apoptosis refers to single cell necrosis in response to injury. An irreversible injury leads to cell death, but the changes described here are not evidence for cellular necrosis. The metabolism of cells is maintained for adaptation, with response to the diuretic and to protein restriction.

A 77-year-old woman has chronic renal failure. Her serum urea nitrogen is 40 mg/dL. She is given a diuretic medication and loses 2 kg (4.4 lb). She reduces the protein in her diet and her serum urea nitrogen decreases to 30 mg/dL. Which of the following terms best describes cellular responses to disease and treatment in this woman? A Adaptation B Apoptosis C Necroptosis D Irreversible injury E Metabolic derangement

*The answer is B.* The cheese-like appearance gives this form of necrosis its name—caseous necrosis. In the lung, tuberculosis and fungal infections are most likely to produce this pattern of tissue injury. Apoptosis involves individual cells, without extensive or localized areas of tissue necrosis. Coagulative necrosis is more typical of ischemic tissue injury. Fat necrosis most often occurs in the breast and pancreas. Fatty change is most often a feature of hepatocyte injury, and the cell integrity is maintained. Gangrene characterizes extensive necrosis of multiple cell types in a body region or organ. Liquefactive necrosis is seen in abscesses or ischemic cerebral injury.

A chest radiograph of an asymptomatic 37-year-old man showed a 3-cm nodule in the middle lobe of the right lung. The nodule was excised with a pulmonary wedge resection, and sectioning showed the nodule to be sharply circumscribed with a soft, white center. Culture of tissue from the nodule grew Mycobacterium tuberculosis. Which of the following pathologic processes has most likely occurred in this nodule? (A) Apoptosis (B) Caseous necrosis (C) Coagulative necrosis (D) Fat necrosis (E) Fatty change (F) Gangrenous necrosis (G) Liquefactive necrosis

*The answer is D.* This biopsy reveals hepatocytes that are distended by large, intracellular fat vacuoles that push nuclei to the periphery. Excessive alcohol consumption induces fat accumulation within hepatocytes, enlarging the liver to as much as three times the normal weight. The amount of fat deposited varies with the amount of alcohol consumed, as well as with the patient's hormonal status and diet. Triglyceride accumulation by itself is not ordinarily damaging, and the condition is fully reversible upon discontinuation of alcohol abuse (abstinence). None of the other choices describe histopathologic features of alcoholic fatty liver.

A liver biopsy is obtained from a 45-year-old man with mildly elevated liver enzymes (aminotransferases). Compared to a normal liver (image on left), the patient's liver biopsy (image on right) shows which of the following histopathologic changes? (A) Bile canaliculi dilated by bile plugs (B) Deposition of lipofuscin in hepatocytes (C) Excess iron deposition in hepatocytes (D) Fat accumulation within hepatocytes (E) Vascular alteration within the liver

*The answer is B.* The grossly cheeselike appearance gives this form of necrosis its name—caseous necrosis. The figure shows amorphous pink acellular material at the upper right surrounded by epithelioid macrophages, and a Langhans giant cell is visible at the upper left. In the lung, tuberculosis and fungal infections are most likely to produce this pattern of tissue injury. Apoptosis involves individual cells, without grossly apparent extensive or localized areas of tissue necrosis. Coagulative necrosis is more typical of ischemic tissue injury. Fat necrosis most often occurs in the breast and pancreas. Fatty change is most often a feature of hepatocyte injury, and the cell integrity is maintained. Gangrene characterizes extensive necrosis of multiple cell types in a body region or organ. Liquefactive necrosis is seen in neutrophilic abscesses or ischemic cerebral injury.

A screening chest radiograph of an asymptomatic 37-year-old man shows a 3-cm nodule in the middle lobe of his right lung. The nodule is excised with a pulmonary wedge resection, and sectioning shows a sharply circumscribed mass with a soft, white center. The microscopic appearance is shown in the figure. The serum interferon gamma release assay is positive. Which of the following pathologic processes has most likely occurred in this nodule? A Apoptosis B Caseous necrosis C Coagulative necrosis D Fat necrosis E Fatty change F Gangrenous necrosis G Liquefactive necrosis

*The answer is A.* This cellular hypertrophy with increase in cell size (not hyperplasia with increase in cell number) accounts for most of the marked increase in size of the uterus during pregnancy. Following pregnancy and reduction in hormonal stimulation, the uterus returns back to its normal size.

A study is performed involving the microscopic analysis of tissues obtained from surgical procedures. Some of these tissues have the microscopic appearance of an increased cell size of multiple cells within the tissue, due to an increase in the amount of cell cytoplasm. Which of the following conditions is most likely to have resulted in this finding? A Uterine myometrium in pregnancy B Female breast at puberty C Liver following partial resection D Ovary following menopause E Cervix with chronic inflammation

*The answer is A.* Irreversible cellular injury is likely to occur when cytoplasmic calcium increases. Calcium can enter cells and also accumulate in mitochondria and endoplasmic reticulum. The excess calcium activates ATPases, phospholipases, proteases, and endonucleases, which injure cell components. Mitochondrial permeability is increased to release cytochrome c, which activates caspases leading to apoptosis. Of the other ions listed, sodium enters the cell, while potassium diffuses out when the sodium pump fails as ATP levels fall; but this is potentially reversible.

A tissue preparation is experimentally subjected to a hypoxic environment. The cells in this tissue begin to swell, and chromatin begins to clump in cell nuclei. ATPases are activated, and ATP production decreases. Which of the following ions accumulating in mitochondria and the cytosol contributes most to these findings and to eventual cell death? A Ca2+ B Cl− C HCO3− D K+ E Na+ F PO43−

Keloids

A year after the laceration of his ear, a 24-year-old African American man develops the lesion shown. What is your diagnosis?

*The answer is A.* Coagulative necrosis is characterized by a massive influx of calcium into the cell. Under normal circumstances, the plasma membrane maintains a steep gradient of calcium ions, whose concentration in interstitial fluids is 10,000 times higher than that inside the cell. Irreversible cell injury damages the plasma membrane, which then fails to maintain this gradient, allowing the infl ux of calcium into the cell. The other choices would most likely be released upon cell death. Diagnosis: Coagulative necrosis

An 82-year-old man has profound bleeding from a peptic ulcer and dies of hypovolemic shock. The liver at autopsy displays centrilobular necrosis. Compared to viable hepatocytes, the necrotic cells contain higher intracellular concentrations of which of the following? (A) Calcium (B) Cobalt (C) Copper (D) Iron (E) Selenium

*The answer is A.* This biopsy of the distal esophagus reveals a glandular surface epithelium with numerous goblet cells (cells with large, pale intracellular vacuoles). A reversible change in cell differentiation in response to injury is termed "metaplasia." Intestinal metaplasia with goblet cells in the distal esophagus is referred to as "Barrett esophagus." This pathologic condition is typically seen in patients with chronic gastroesophageal reflux. The lesion is characterized histologically by intestine-like epithelium, composed of goblet cells incompletely intestinalized gastric mucosa. Barrett esophagus is more resistant to peptic juices than normal squamous epithelium and appears to be an adaptive mechanism that serves to limit the harmful effects of gastroesophageal reflux. None of the other choices describe intestinal metaplasia.

An endoscopic biopsy of the distal esophagus is obtained from a 50-year-old man with a history of chronic epigastric pain (heartburn). How does the histologic appearance of the patient's biopsy (image on right) differ from that of a normal esophagus (image on left)? (A) The cell type of the surface epithelium has changed. (B) The esophageal mucosa has been eroded. (C) There has been a neoplastic change in the epithelium. (D) There is a foreign material within the epithelium. (E) There is an infectious process within the epithelium.

*The answer is D.* Reduction in oxidative phosphorylation leads to reduction in synthesis of ATP and diminished activity of the plasma membrane sodium pump, which maintains high intracellular potassium concentration. Loss of ATP leads to efflux of intracellular potassium, while net influx of sodium and water promote cell swelling. A marked rise in plasma potassium can indicate significant cell damage or death (such as skeletal muscle crush injury or hemolysis). When cells are not consuming glucose via oxidative metabolism, the glucose is metabolized via other pathways, and glucose is maintained within normal ranges. Though cell membranes are composed of lipid, dysfunction or disruption of those membranes does not significantly alter plasma lipid concentrations.

An experimental drug administered to a tissue preparation is found to inhibit cellular oxidative phosphorylation when given in high doses, and ATP production drops to 5% of normal. Cell membrane function is diminished. Which of the following substances is most likely to be present at increased concentration in culture fluid bathing the tissue? A Calcium B Glucose C Ketones D Potassium E Sodium

*The answer is B.* Endometrial hyperplasia most often is caused by excess estrogen without progesterone.

An oophorectomized monkey is treated with high doses of estrogen. Which of the following changes is most likely to occur in the endometrium after 1 year of treatment? (A) Atrophy (B) Hyperplasia (C) Hypertrophy (D) Hypoplasia (E) Metaplasia

Irreversible injured because of the missing and fading nuclei (karyolysis).

Are these liver cells normal, reversibly injured, or irreversibly injured and why?

*The answer is C.* The appearance of lipid vacuoles in many of the hepatocytes is characteristic of fatty change (steatosis) of the liver. Abnormalities in lipoprotein metabolism can lead to steatosis. Alcohol is a hepatotoxin that produces hepatic steatosis. Decreased serum albumin levels and increased prothrombin time suggest alcohol-induced hepatocyte damage. Substance abuse with heroin produces few organ-specific pathologic findings. Exercise has little direct effect on hepatic function. Smoking directly damages lung tissue, but has no direct effect on the liver. Aspirin has a significant effect on platelet function, but not on hepatocytes.

At autopsy, a 40-year-old man has an enlarged (2200 g) liver with a yellow cut surface. The microscopic appearance of this liver is shown in the figure. Before death, the man's total serum cholesterol and triglyceride levels were normal, but he had a decreased serum albumin concentration and increased prothrombin time. Which of the following activities most likely led to these findings? (A) Injecting heroin (B) Playing basketball (C) Drinking beer (D) Smoking cigarettes (E) Ingesting aspirin

Coagulative Necrosis

Based on the morphology present in this kidney, what form of necrosis is most likely the cause of this lesion?

*The answer is C.* Hyperplasia is defined as an increase in the number of cells in an organ or tissue. Like hypertrophy (choice D), it is often a response to trophic signals or increased functional demand and is commonly a normal process. Erythroid hyperplasia is typically seen in people living at high altitude. Low oxygen tension evokes the production of erythropoietin, which promotes the survival and proliferation of erythroid precursors in the bone marrow. The cellular and molecular mechanisms that are responsible for hyperplasia clearly relate to the control of cell proliferation (i.e., cell cycle). None of the other choices describe increased numbers of cells. Diagnosis: Erythropoiesis, hyperplasia

Bone marrow cells from an organ donor are cultured in vitro at 37°C in the presence of recombinant erythropoietin. A photomicrograph of a typical "burst-forming unit" is shown in the image. This colony, committed to the erythrocyte pathway of differentiation, represents an example of which of the following physiologic adaptations to transmembrane signaling? (A) Atrophy (B) Dysplasia (C) Hyperplasia (D) Hypertrophy (E) Metaplasia

*The answer is A.* Breast involution following lactation is an example of physiologic atrophy.

Breast involution following lactation is an example of: A. Physiologic atrophy B. Pathologic atrophy

*The answer is C.* In the intrinsic pathway of apoptosis, caspases cause cytochrome c release.

During an experimental study, an investigator finds that the regulation of cell cycle and programmed cell death may be initiated by the mitochondrion. The interaction of the mitochondrion with the activation of the caspase family of proteases and subsequent apoptosis is most likely mediated by which of the following? (A) Calcium release (B) cAMP production (C) Cytochrome c release (D) GTP binding (E) Nitric oxide release

*The answer is B.* Embryonic stem cells are pluripotent.

Embryonic stem cells are multipotent. A. True B. False

*The answer is A.* Fibrinoid necrosis occurs in the arteriolar walls in malignant hypertension.

Fibrinoid necrosis: A Occurs in the arteriolar walls in malignant hypertension. B Appears blue on haematoxylin and eosin staining. C Is usually due to clostridial infection. D Is caused by enzymatic lysis of adipose tissue. E Is the characteristic appearance seen in tuberculosis.

*The answer is B.* The drug acetaminophen can be converted to toxic metabolites in this manner. Glutathione in the cytosol helps to reduce cellular injury from many toxic metabolites and free radicals. ADP is converted to ATP by oxidative and glycolytic cellular pathways to provide energy that drives cellular functions, and a reduction in ATP leaves the cell vulnerable to injury. NADPH oxidase generates superoxide, which is used by neutrophils in killing bacteria. Nitric oxide synthase in macrophages produces nitric oxide, which aids in destroying organisms undergoing phagocytosis. Protein synthesis in cells depends on mRNA for longer survival and recovery from damage caused by free radicals. Failure of the sodium pump leads to increased cytosolic sodium and cell swelling with injury.

In an experiment, a large amount of a drug is administered to experimental organisms and is converted by cytochrome P-450 to a toxic metabolite. Accumulation of this metabolite leads to increased intracellular lipid peroxidation. Depletion of which of the following intracellular substances within the cytosol exacerbates this form of cellular injury by this mechanism? A ADP B Glutathione C NADPH oxidase D Nitric oxide synthase E mRNA F Sodium

*The answer is B.* The body has intracellular mechanisms that prevent damage from free radicals generated by exposure to x-rays. Glutathione peroxidase reduces such injury by catalyzing the breakdown of hydrogen peroxide. Endonucleases damage DNA in nuclear chromatin. Lactate dehydrogenase is present in a variety of cells, and its elevation in the serum is an indicator of cell injury and death. Phospholipases decrease cellular phospholipids and promote cell membrane injury. Proteases can damage cell membranes and cytoskeletal proteins.

In an experiment, metabolically active cells are subjected to radiant energy in the form of x-rays. This results in cell injury caused by hydrolysis of water. Which of the following intracellular enzymes helps to protect the cells from this type of injury? A Endonuclease B Glutathione peroxidase C Lactate dehydrogenase D Phospholipase E Protease

*The answer is D.* This photomicrograph shows coagulative necrosis.

In the above photomicrograph labeled A, what pathologic change is seen in the myocardium? (A photomicrograph of normal myocardium is included for comparison) A. Reversible cell injury B. Cardiac hypertrophy C. Liquefactive necrosis D. Coagulative necrosis E. Fatty cell change

*The answer is C.* Cells would die by nerosis by the removal of heart muscle cells damaged by oxygen depletion following cardiac infarction.

In which of the following situations would cells die by necrosis, not apoptosis? a) Removal of cells with damaged DNA that cannot be repaired. b) Removal of developing neurons that fail to make profitable connections with other cells. c) Removal of heart muscle cells damaged by oxygen depletion following cardiac infarction. d) Removal of virus infected cells.

*The answer is B.* Irreversible ischemic injury in the brain would present morphologically as liquefactive necrosis.

Irreversible ischemic injury in the brain would present morphologically as coagulative necrosis. A. True B. False

*The answer is E.* Adaptive responses to sublethal cell injury include atrophy, hypertrophy, hyperplasia, metaplasia, and dysplasia. Metaplasia is the conversion of one cell differentiation pathway to another. In this autopsy specimen, the normal stratified squamous epithelium of the esophagus (on the left) has been replaced by columnar epithelium with goblet cells (on the right) as a result of chronic injury. The distal esophagus is said to exhibit "intestinal metaplasia." This disorder (Barrett esophagus) typically occurs in the lower third of the esophagus. Complete intestinal metaplasia with Paneth cells and absorptive cells may also occur. Barrett esophagus is more resistant to peptic juice than normal squamous epithelium and appears to be an adaptive mechanism that serves to limit the harmful effects of acid reflux. None of the other cellular adaptations describe histopathologic findings in patients with Barrett esophagus.

Microscopic examination of the distal portion of an autopsy specimen provided reveals intestine-like glandular epithelium with goblet cells. These histopathologic findings are associated with which of following adaptations to chronic persistent cell injury? (A) Atrophy (B) Dysplasia (C) Hyperplasia (D) Hypertrophy (E) Metaplasia

*The answer is E.* The pressure load on the left ventricle results in an increase in myofilaments in the existing myofibers. The result of continued stress from hypertension is eventual heart failure with decreased contractility, but the cells do not decrease in size. Metaplasia of muscle does not occur, although loss of muscle occurs with aging as myofibers are replaced by fibrous tissue and adipose tissue. Hemosiderin deposition in the heart is a pathologic process resulting from increased iron stores in the body.

On a routine visit to the physician, an otherwise healthy 51-year-old man has a blood pressure of 150/95 mm Hg. If his hypertension remains untreated for years, which of the following cellular alterations would most likely be seen in his myocardium? (A) Atrophy (B) Hyperplasia (C) Metaplasia (D) Hemosiderosis (E) Hypertrophy

*The answer is A.*The onset of menstruation is orderly, programmed cell death (apoptosis) through hormonal stimuli, an example of the intrinsic (mitochondrial) apoptotic pathway. As hormone levels drop, the endometrium breaks down, sloughs off, and then regenerates. With cellular atrophy, there is often no visible necrosis, but the tissues shrink, something that occurs in the endometrium after menopause. Caseous necrosis is typical of granulomatous inflammation, resulting most commonly from mycobacterial infection. Heterophagocytosis is typified by the clearing of an area of necrosis through macrophage ingestion of the necrotic cells. Liquefactive necrosis can occur in any tissue after acute bacterial infection or in the brain after ischemia.

On day 28 of her menstrual cycle, a 23-year-old woman experiences onset of menstrual bleeding that lasts for 6 days. She has had regular cycles since menarche. Which of the following processes most likely occurs in her endometrial cells to initiate the onset of menstrual bleeding? A Apoptosis B Atrophy C Caseous necrosis D Heterophagocytosis E Liquefactive necrosis

*The answer is B.* You would see more cells in hyperplasia.

The prostate at top is normal, the one at the bottom has an adaptation: this is BPH: benign prostatic HYPERPLASIA. Under the microscope, we see big cells but not more cells. A. True B. False

*The answer is A.* This person most likely had a heart attack, which results in coagulative necrosis.

This figure illustrates the microscopic appearance of the heart of a 56-year-old man who died after a 24-hour hospitalization for severe "crushing" chest pain complicated by hypotension and pulmonary edema. The type of necrosis shown is best described as: (A) Coagulative. (B) Liquefactive. (C) Caseous.

*The answer is C.* This biopsy reveals skeletal muscle myofibers with peripheral nuclei surrounded by chronic inflammatory cells (numerous basophilic nuclei). The patient may have been an elderly man recovering from the "flu" who complains of marked fatigability. He may report that he cannot climb stairs two at a time as he used to. He may also describe pain in the thighs. Inflammatory myopathies are thought to have an autoimmune origin. The most common morphologic characteristics in the inflammatory myopathies are (1) inflammatory cells, (2) necrosis and phagocytosis of muscle fibers, (3) a mixture of regenerating and atrophic fibers, and (4) fibrosis. Polymyositis is one example of an inflammatory myopathy that is related to direct muscle cell damage produced by cytotoxic T cells. Healthy muscle fibers are initially surrounded by CD8+ T lymphocytes and macrophages, after which muscle fibers degenerate. The patient's biopsy does not show cardiac or smooth muscle (choices A, B, D, and E).

You examine a slide with a surgical pathologist at a double-headed microscope (shown in the image) and are asked to describe what you see. Which of the following best describes this tissue and the response to injury? (A) Cardiac muscle, fibrosis (B) Cardiac muscle, necrosis (C) Skeletal muscle, inflammation (D) Smooth muscle, fibrosis (E) Smooth muscle, inflammation

*The answer is B.* Though cells with damaged DNA are killed by apoptosis if the damage cannot be repaired, most forms of damage, such as physical damage, and damage produced by oxygen deprivation produce necrosis. Here, membranes are ruptured and cell contents leak out causing inflammation in the surrounding tissue. Necrotic cell death is an accidental, not a regulated, process ==an OOPS! Death

You have just burnt your finger on the stove. The cells die by apoptosis. True or False? a) True b) False


Set pelajaran terkait

Science Chapter 12 *ALL (SUMMERS) ANSWERS PLUS THE LAST QUESTION*

View Set

Chapter 1: Five Foundations of Economics

View Set

Contemporary Health Issues Chapters 14-17

View Set

Chapter 1 (McGraw Hill Connect Questions)

View Set

Basics of Coordinate Systems and Map Projections

View Set

AP History Chapter 16 MindTap Exercises 16.8-16.13

View Set

CH 9: Starting Out With C++ CHECKPOINT QUESTIONS

View Set

Business Mathematics - Chapter 13

View Set